m
 
(5 intermediate revisions by the same user not shown)
Line 14: Line 14:
 
Automatic Control (AC)
 
Automatic Control (AC)
  
Question 3: Optimization   WORK IN PROGRESS
+
Question 3: Optimization
 
</font size>
 
</font size>
  
Line 21: Line 21:
 
----
 
----
 
----
 
----
:Student answers and discussions for [[QE2012_AC-3_ECE580-1|Part 1]],[[QE2012_AC-3_ECE580-2|2]],[[QE2012_AC-3_ECE580-2|3]],[[QE2012_AC-3_ECE580-4|4]],[[QE2012_AC-3_ECE580-5|5]]
+
:Student answers and discussions for [[QE2013_AC-3_ECE580-1|Part 1]],[[QE2013_AC-3_ECE580-2|2]],[[QE2013_AC-3_ECE580-3|3]],[[QE2013_AC-3_ECE580-4|4]],[[QE2013_AC-3_ECE580-5|5]]
 
----
 
----
 
'''1.(20 pts) In some of the optimization methods, when minimizing a given function f(x), we select an intial guess <math>x^{(0)}</math> and a real symmetric positive definite matrix <math>H_{0}</math>.  Then we computed <math>d^{(k)} = -H_{k}g^{(k)}</math>, where <math>g^{(k)} = \nabla f( x^{(k)} )</math>, and <math>x^{(k+1)} = x^{(k)} + \alpha_{k}d^{(k)}</math>, where'''  
 
'''1.(20 pts) In some of the optimization methods, when minimizing a given function f(x), we select an intial guess <math>x^{(0)}</math> and a real symmetric positive definite matrix <math>H_{0}</math>.  Then we computed <math>d^{(k)} = -H_{k}g^{(k)}</math>, where <math>g^{(k)} = \nabla f( x^{(k)} )</math>, and <math>x^{(k+1)} = x^{(k)} + \alpha_{k}d^{(k)}</math>, where'''  
Line 31: Line 31:
 
<math> f(x) = \frac{1}{2} x^{T} Qx - x^{T}b+c, Q = Q^{T} > 0. </math>
 
<math> f(x) = \frac{1}{2} x^{T} Qx - x^{T}b+c, Q = Q^{T} > 0. </math>
 
<br><br>
 
<br><br>
'''(i)(10 pts) Find a closed form expression for <math>\alpha_k</math> in terms of <math>Q, H_k, g^{(k)}, and d^{(k)}; </math>'''
+
'''(i)(10 pts) Find a closed form expression for <math>\alpha_k</math> in terms of <math>Q, H_k, g^{(k)}</math>, and <math>d^{(k)}; </math>'''
 
<br>  
 
<br>  
 
'''(ii)(10 pts) Give a sufficient condition on <math>H_k</math> for <math>\alpha_k</math> to be positive.'''
 
'''(ii)(10 pts) Give a sufficient condition on <math>H_k</math> for <math>\alpha_k</math> to be positive.'''
  
:'''Click [[QE2012_AC-3_ECE580-1|here]] to view [[QE2012_AC-3_ECE580-1|student answers and discussions]]'''
+
:'''Click [[QE2013_AC-3_ECE580-1|here]] to view [[QE2013_AC-3_ECE580-1|student answers and discussions]]'''
 
----
 
----
  
Line 50: Line 50:
 
'''Find the probability that the mutation operation destroys the schema, where the probability of random change of each symbol of the chromosome is <math>p_m = 0.1</math> independently.'''
 
'''Find the probability that the mutation operation destroys the schema, where the probability of random change of each symbol of the chromosome is <math>p_m = 0.1</math> independently.'''
  
:'''Click [[QE2012_AC-3_ECE580-2|here]] to view [[QE2012_AC-3_ECE580-2|student answers and discussions]]'''
+
:'''Click [[QE2013_AC-3_ECE580-2|here]] to view [[QE2013_AC-3_ECE580-2|student answers and discussions]]'''
  
 
----
 
----
Line 74: Line 74:
 
'''[(ii) (10 pts)] Construct the dual program of the linear program above and solve it. '''
 
'''[(ii) (10 pts)] Construct the dual program of the linear program above and solve it. '''
  
:'''Click [[QE2012_AC-3_ECE580-3|here]] to view [[QE2012_AC-3_ECE580-3|student answers and discussions]]'''  
+
:'''Click [[QE2013_AC-3_ECE580-3|here]] to view [[QE2013_AC-3_ECE580-3|student answers and discussions]]'''  
 
----
 
----
'''Problem 4. (20pts) Use any simplex method to solve the following linear program. '''  
+
'''Problem 4. (20pts) Consider the following model of a discrete-time system, '''  
 
+
<br>
            <span class="texhtml">''Maximize''</span>'''    <span class="texhtml">''x''<sub>1</sub> + 2''x''<sub>2</sub></span>'''
+
<math> x(k+1) = x(k) + 2u(k), x(0) = 3, 0 \le k \le 2</math>
        <span class="texhtml">''S'ubject to''</span>'''    <math>-2x_1+x_2 \le 2</math>'''
+
<br>
                        <math>x_1-x_2 \ge -3</math>
+
'''Use the Lagrange multiplier approach to calculate the optimal control sequence'''
                        <math>x_1 \le -3</math>
+
<br>
                        <math>x_1 \ge 0, x_2 \ge 0.</math>
+
{u(0), u(1), u(2)}
 +
<br>
 +
''' that transfers the initial state x(0) to x(3) = 9 while minimizing the performance index'''
 +
<br>
 +
<math> J = \frac{1}{2} \sum_{k=0}^2 u(k)^2 = \frac{1}{2}u^Tu. </math>
 +
<br>
 +
''' Hint: Use the system model to obtain a constraint of the form, <math>Au = b, A \in R^{1 \times 3}. </math>'''
  
:'''Click [[QE2012_AC-3_ECE580-4|here]] to view [[QE2012_AC-3_ECE580-4|student answers and discussions]]'''
+
:'''Click [[QE2013_AC-3_ECE580-4|here]] to view [[QE2013_AC-3_ECE580-4|student answers and discussions]]'''
 
----
 
----
  
<br> '''Problem 5.(20pts) Solve the following problem:'''  
+
<br> '''Problem 5. (20pts) Find minimizers and maximizers of the function, '''  
 
+
<br>
            <span class="texhtml">''Minimize''</span>'''    <math>-x_1^2 + 2x_2</math>'''
+
<math> f(x) = (a^Tx)(b^Tx), x \in R^3, </math>
        <span class="texhtml">''Subject to''</span>'''    <math>x_1^2+x_2^2 \le 1</math>'''
+
<br>
                        <math> x_1 \ge 0</math>
+
'''subject to'''
                        <math>x_2 \ge 0</math>
+
<br>
 
+
<math> x_1 + x_2 = 0 </math>
'''(i)(10pts) Find the points that satisfy the KKT condition.'''  
+
<br>
 
+
<math> x_2 + x_3 = 0, </math>  
 
+
<br>
 
+
'''where'''
<br> '''(ii)(10pts)Apply the SONC and SOSC to determine the nature of the critical points from the previous part.'''
+
<br>
 +
<math>a = \begin{bmatrix}
 +
  0  \\
 +
  1 \\
 +
  0
 +
\end{bmatrix}</math> and <math>b = \begin{bmatrix}
 +
  1  \\
 +
  0 \\
 +
  1
 +
\end{bmatrix}</math>
  
:'''Click [[QE2012_AC-3_ECE580-5|here]] to view [[QE2012_AC-3_ECE580-5|student answers and discussions]]'''
+
:'''Click [[QE2013_AC-3_ECE580-5|here]] to view [[QE2013_AC-3_ECE580-5|student answers and discussions]]'''
 
----
 
----
 
[[ECE_PhD_Qualifying_Exams|Back to ECE QE page]]
 
[[ECE_PhD_Qualifying_Exams|Back to ECE QE page]]

Latest revision as of 12:23, 25 March 2015


ECE Ph.D. Qualifying Exam

Automatic Control (AC)

Question 3: Optimization

August 2013



Student answers and discussions for Part 1,2,3,4,5

1.(20 pts) In some of the optimization methods, when minimizing a given function f(x), we select an intial guess $ x^{(0)} $ and a real symmetric positive definite matrix $ H_{0} $. Then we computed $ d^{(k)} = -H_{k}g^{(k)} $, where $ g^{(k)} = \nabla f( x^{(k)} ) $, and $ x^{(k+1)} = x^{(k)} + \alpha_{k}d^{(k)} $, where
$ \alpha_{k} = arg\min_{\alpha \ge 0}f(x^{(k)} + \alpha d^{(k)}) . $
Suppose that the function we wish to minimize is a standard quadratic of the form,
$ f(x) = \frac{1}{2} x^{T} Qx - x^{T}b+c, Q = Q^{T} > 0. $

(i)(10 pts) Find a closed form expression for $ \alpha_k $ in terms of $ Q, H_k, g^{(k)} $, and $ d^{(k)}; $
(ii)(10 pts) Give a sufficient condition on $ H_k $ for $ \alpha_k $ to be positive.

Click here to view student answers and discussions

Problem 2. (20 pts) [(i) (10 pts)] Consider the one-point crossover of a chromosome in the schema
H = * 1 * 0 1 0 *
where the probability that a chromosome is chosen for crossover is $ p_c = 0.5. $ Find an upper bound on the probability that a chromosome from H will be destroyed by the one-point crossover.

[(ii) (10 pts)] Consider a chromosome in the schema
H = * 1 * 0 * * *
Find the probability that the mutation operation destroys the schema, where the probability of random change of each symbol of the chromosome is $ p_m = 0.1 $ independently.

Click here to view student answers and discussions

Problem 3. (20 pts) [(i) (10 pts)] Convert the following optimization problem into a linear programming problem and solve it;
maximize $ -|x_1| -|x_2| -|x_3| $
subject to
$ \begin{bmatrix} 1 & 1 &-1 \\ 0 & -1 & 0 \end{bmatrix} \begin{bmatrix} x_1 \\ x_2 \\ x_3 \end{bmatrix} = \begin{bmatrix} 2 \\ 1 \end{bmatrix} . $

[(ii) (10 pts)] Construct the dual program of the linear program above and solve it.

Click here to view student answers and discussions

Problem 4. (20pts) Consider the following model of a discrete-time system,
$ x(k+1) = x(k) + 2u(k), x(0) = 3, 0 \le k \le 2 $
Use the Lagrange multiplier approach to calculate the optimal control sequence
{u(0), u(1), u(2)}
that transfers the initial state x(0) to x(3) = 9 while minimizing the performance index
$ J = \frac{1}{2} \sum_{k=0}^2 u(k)^2 = \frac{1}{2}u^Tu. $
Hint: Use the system model to obtain a constraint of the form, $ Au = b, A \in R^{1 \times 3}. $

Click here to view student answers and discussions


Problem 5. (20pts) Find minimizers and maximizers of the function,
$ f(x) = (a^Tx)(b^Tx), x \in R^3, $
subject to
$ x_1 + x_2 = 0 $
$ x_2 + x_3 = 0, $
where
$ a = \begin{bmatrix} 0 \\ 1 \\ 0 \end{bmatrix} $ and $ b = \begin{bmatrix} 1 \\ 0 \\ 1 \end{bmatrix} $

Click here to view student answers and discussions

Back to ECE QE page

Alumni Liaison

Ph.D. 2007, working on developing cool imaging technologies for digital cameras, camera phones, and video surveillance cameras.

Buyue Zhang